Answered step by step
Verified Expert Solution
Link Copied!

Question

1 Approved Answer

Your wholesale auto parts company is considering expanding its territory to Illinois. To do so, you would need to hire sales reps to cover the

Your wholesale auto parts company is considering expanding its territory to Illinois. To do so, you would need to hire sales reps to cover the new territory and are considering how many reps are optimal for your company.

If the margin (sales revenue minus COGS divided by sales) is 30%, and sales reps cost $9,000 per month (including salary, benefits, and travel expenses), how many sales reps should you employ? Based on that level of sales reps, how much incremental (monthly) profit will you generate by selling in Illinois?

Number of Sales Reps Total monthly sales revenue in Illinois
0 -
1 55,000
2 97,000
3 139,000
4 168,000
5 192,000
6 214,000
7 230,000
8 246,000
9 260,000
10

270,000

Contribution is revenue minus cost of goods sold.

Sales Rep Cost is the cost per month multiplied by the number of sales reps.

Net benefits is the Contribution minus the sales rep costs.

Marginal benefit is the change in benefits by adding one more sales rep. In other words, how much do benefits go up by increasing to that sales level.

Marginal cost is the change in sales rep costs by adding one more salesperson.

Marginal net benefit is the difference between marginal benefit and marginal cost.

Step by Step Solution

There are 3 Steps involved in it

Step: 1

blur-text-image

Get Instant Access to Expert-Tailored Solutions

See step-by-step solutions with expert insights and AI powered tools for academic success

Step: 2

blur-text-image

Step: 3

blur-text-image

Ace Your Homework with AI

Get the answers you need in no time with our AI-driven, step-by-step assistance

Get Started

Recommended Textbook for

Foundations In Personal Finance

Authors: Dave Ramsey

College Edition

1936948001, 978-1936948000

More Books

Students also viewed these Finance questions

Question

=+4. What do you think?

Answered: 1 week ago